- Mon Jan 20, 2014 12:00 am
#38160
Complete Question Explanation
(The complete setup for this game can be found here: lsat/viewtopic.php?t=14979)
The correct answer choice is (D)
For the first time ever, test-makers decided to include two Rule Substitution questions in the Logic Games section, of which this is the second one (the first one was Question #5 in Game 1). Considering their growing popularity, tackling such questions is no longer merely optional: it is imperative to have a workable strategy in place.
The question stem requires you to replace the fourth rule of the game, which created the following relationship:
Thankfully, the rule only concerns the order in which the books are being discussed (regardless of whether they are summarized or not). Without this rule, we are left with the following sequencing relationship:
Since Additional Effects are easier to spot, look for answer choices that contain such conditions first. You should immediately notice, for instance, that answer choices (A), (B), and (E) add conditions extraneous to the original rule set. Answer choice (A) stipulates that T must be discussed third, which is not necessarily true (T can also be discussed second). Likewise, answer choice (B) requires T to be discussed earlier than F, which is not an inference we can make using our original diagram. Finally, answer choice (E) requires F to be discussed third. While F must be among the first three books discussed, it is not necessarily third. Thus, we can safely eliminate these Additional Effects answer choices.
We are left with answer choices (C) and (D). According to answer choice (C), K and R are among the last three books discussed, and F is among the first three books discussed. Both conditions are true given our original diagram; however, they do not have the exact same effect on the order in which the books are discussed. Since K and R are only required to be among the last three (not two) books discussed, O is no longer constrained to occupy the fourth position. Indeed, O can be fourth, fifth, or sixth—in violation of our original diagram! Therefore, answer choice (C) contains a Partial Match, and must be eliminated.
Answer choice (D): This is the correct answer choice. Both conditions in this answer choice—K and R are discussed consecutively, and O is discussed fourth—are consistent with the original diagram. Answer choice (D) does not therefore contain an Additional Effects rule. Answer choice (D) is not a Partial Match either: the conditions force K and R to be the last two books discussed, because if they were discussed earlier than O, we would not be able to accommodate the N T O sequence. Thus, the effect is identical, and this is the correct answer choice.
(The complete setup for this game can be found here: lsat/viewtopic.php?t=14979)
The correct answer choice is (D)
For the first time ever, test-makers decided to include two Rule Substitution questions in the Logic Games section, of which this is the second one (the first one was Question #5 in Game 1). Considering their growing popularity, tackling such questions is no longer merely optional: it is imperative to have a workable strategy in place.
The question stem requires you to replace the fourth rule of the game, which created the following relationship:
Thankfully, the rule only concerns the order in which the books are being discussed (regardless of whether they are summarized or not). Without this rule, we are left with the following sequencing relationship:
- N T O
Since Additional Effects are easier to spot, look for answer choices that contain such conditions first. You should immediately notice, for instance, that answer choices (A), (B), and (E) add conditions extraneous to the original rule set. Answer choice (A) stipulates that T must be discussed third, which is not necessarily true (T can also be discussed second). Likewise, answer choice (B) requires T to be discussed earlier than F, which is not an inference we can make using our original diagram. Finally, answer choice (E) requires F to be discussed third. While F must be among the first three books discussed, it is not necessarily third. Thus, we can safely eliminate these Additional Effects answer choices.
We are left with answer choices (C) and (D). According to answer choice (C), K and R are among the last three books discussed, and F is among the first three books discussed. Both conditions are true given our original diagram; however, they do not have the exact same effect on the order in which the books are discussed. Since K and R are only required to be among the last three (not two) books discussed, O is no longer constrained to occupy the fourth position. Indeed, O can be fourth, fifth, or sixth—in violation of our original diagram! Therefore, answer choice (C) contains a Partial Match, and must be eliminated.
Answer choice (D): This is the correct answer choice. Both conditions in this answer choice—K and R are discussed consecutively, and O is discussed fourth—are consistent with the original diagram. Answer choice (D) does not therefore contain an Additional Effects rule. Answer choice (D) is not a Partial Match either: the conditions force K and R to be the last two books discussed, because if they were discussed earlier than O, we would not be able to accommodate the N T O sequence. Thus, the effect is identical, and this is the correct answer choice.
You do not have the required permissions to view the files attached to this post.